Telescopic Series

Calculus Level 4

n = 1 1 ( n + 1 ) ( n + 2 ) ( n + 3 ) ( n + 4 ) = a b \sum _{ n=1 }^{ \infty }{ \frac { 1 }{ (n+1)(n+2)(n+3)(n+4) } } =\frac { a }{ b } a a and b b are co-prime integers. Find a + b a+b .


The answer is 73.

This section requires Javascript.
You are seeing this because something didn't load right. We suggest you, (a) try refreshing the page, (b) enabling javascript if it is disabled on your browser and, finally, (c) loading the non-javascript version of this page . We're sorry about the hassle.

2 solutions

Discussions for this problem are now closed

Kuang-Lin Pan
Feb 28, 2015

Inspired by the way of canceling terms: n = 1 1 ( n + 1 ) ( n + 2 ) = n = 1 ( 1 n + 1 1 n + 2 ) = ( 1 2 2 3 ) + ( 2 3 3 4 ) + ( 3 4 4 5 ) + = 1 2 , \sum_{n=1}^\infty \frac{1}{(n+1)(n+2)}\\ =\sum_{n=1}^\infty \left(\frac{1}{n+1}-\frac{1}{n+2}\right)\\ =\left(\frac{1}{2}-\frac{2}{3}\right)+\left(\frac{2}{3}-\frac{3}{4}\right)+\left(\frac{3}{4}-\frac{4}{5}\right)+\ldots =\frac{1}{2} , (Cancel the repeated terms with opposite signs and the last term that is not canceled converges to 0 0 )

We can have n = 1 1 ( n + 1 ) ( n + 2 ) ( n + 3 ) ( n + 4 ) = 1 3 × n = 1 [ 1 ( n + 1 ) ( n + 2 ) ( n + 3 ) 1 ( n + 2 ) ( n + 3 ) ( n + 4 ) ] \sum_{n=1}^\infty \frac{1}{(n+1)(n+2)(n+3)(n+4)}\\ =\frac{1}{3}\times\sum_{n=1}^\infty \left[\frac{1}{(n+1)(n+2)(n+3)}-\frac{1}{(n+2)(n+3)(n+4)}\right]\\ Using similar strategy, the expression equals to 1 3 × 1 2 3 4 = 1 72 \frac{1}{3}\times\frac{1}{2 \cdot 3 \cdot 4}=\frac{1}{72} , it turns out that a + b = 73 a+b=73 .

Ah, why didn't I think of that earlier?! This makes the problem a one-liner and is much more elegant than my approach. Upvoted. :)


And just for the record, this "strategy" is called a telescoping sum where all the subsequent terms of the sum gets cancelled out.

Prasun Biswas - 6 years, 3 months ago

Nice Solution :D

Paul Ryan Longhas - 6 years, 3 months ago

Nice solution,anyway,can you teach me about calculus?

Frankie Fook - 6 years, 3 months ago
Bhargav Upadhyay
Feb 27, 2015

n = 1 1 ( n + 1 ) ( n + 2 ) ( n + 3 ) ( n + 4 ) 1 ( n + 1 ) ( n + 2 ) ( n + 3 ) ( n + 4 ) = 1 6 ( n + 1 ) + 1 2 ( n + 2 ) + 1 2 ( n + 3 ) + 1 6 ( n + 4 ) = n = 1 1 6 ( 1 ( n + 1 ) 1 ( n + 4 ) ) n = 1 1 2 ( 1 ( n + 2 ) 1 ( n + 3 ) ) = 1 6 ( 1 2 + 1 3 + 1 4 ) 1 2 ( 1 3 ) = 1 6 ( 13 12 1 ) = 1 72 a = 1 , b = 72 a + b = 73 \sum _{ n=1 }^{ \infty }{ \frac { 1 }{ (n+1)(n+2)(n+3)(n+4) } } \\ \frac { 1 }{ (n+1)(n+2)(n+3)(n+4) } \quad =\quad \frac { 1 }{ 6(n+1) } +\frac { -1 }{ 2(n+2) } +\frac { 1 }{ 2(n+3) } +\frac { -1 }{ 6(n+4) } \\ =\sum _{ n=1 }^{ \infty }{ \frac { 1 }{ 6 } } (\frac { 1 }{ (n+1) } -\frac { 1 }{ (n+4) } )\quad -\sum _{ n=1 }^{ \infty }{ \frac { 1 }{ 2 } } (\frac { 1 }{ (n+2) } -\frac { 1 }{ (n+3) } )\\ =\frac { 1 }{ 6 } (\frac { 1 }{ 2 } +\frac { 1 }{ 3 } +\frac { 1 }{ 4 } )\quad -\quad \frac { 1 }{ 2 } (\frac { 1 }{ 3 } )\quad =\quad \frac { 1 }{ 6 } (\frac { 13 }{ 12 } -1)\quad =\quad \frac { 1 }{ 72 } \\ \therefore a=1,\quad b=72\\ \therefore a+b\quad =\quad 73\\

Since I'm lazy to use partial fractional decomposition method, I used a shorter approach. Notice that ( n + 1 ) ( n + 4 ) (n+1)(n+4) and ( n + 2 ) ( n + 3 ) (n+2)(n+3) when expanded have every term same except the constant term at the end. The constants differ by 2 2 . We use this fact and first break the general term as follows:

f ( n ) = 1 ( n + 1 ) ( n + 2 ) ( n + 3 ) ( n + 4 ) f ( n ) = 1 2 ( 1 ( n + 1 ) ( n + 4 ) 1 ( n + 2 ) ( n + 3 ) ) f(n)=\frac{1}{(n+1)(n+2)(n+3)(n+4)}\\ \implies f(n)=\frac{1}{2}\cdot \left(\frac{1}{(n+1)(n+4)}-\frac{1}{(n+2)(n+3)}\right)

Now, we can simply break each of the two fractions inside f ( n ) f(n) by introducing special zero values in the numerator and the whole general term breaks as:

f ( n ) = 1 2 { 1 3 ( 1 n + 1 1 n + 4 ) ( 1 n + 2 1 n + 3 ) } f(n)=\frac{1}{2}\left\{ \frac{1}{3}\left(\frac{1}{n+1}-\frac{1}{n+4}\right)-\left(\frac{1}{n+2}-\frac{1}{n+3}\right)\right\}

Rearranging the terms gives us the same as you evaluated using partial fractions. Although I think that rearranging it is redundant and one can calculate the sum just by telescoping each of the smaller sums of the general term.

Prasun Biswas - 6 years, 3 months ago

Hey really good observation... I find it very quick and easy to do partial fraction of first order polynomial so gone for it..

Bhargav Upadhyay - 6 years, 3 months ago

0 pending reports

×

Problem Loading...

Note Loading...

Set Loading...